Sample Exam 1 Flashcards

1
Q

Which of the following advisors must register with the U.S. Securities and Exchange Commission (SEC)?

A) An advisor who sells insurance products

B) An advisor who is registered with his or her state securities regulator

C) An advisor who manages over $100 million in assets

D) An advisor who declines to be regulated by FINRA

A

The correct answer is (C).
Advisors with at least $100 million in assets under management must register with the SEC. Advisors with less than $100 million in assets under management must register with their respective state regulator.

How well did you know this?
1
Not at all
2
3
4
5
Perfectly
2
Q

For a call option contract, the price at which the option holder can buy the underlying security is called the

A) premium.

B) exercise price.

C) intrinsic value.

D) time value.

A

The correct answer is (B).
The price at which an option holder buys the underlying stock is called the exercise price or the strike price.

How well did you know this?
1
Not at all
2
3
4
5
Perfectly
3
Q

Harold would like to purchase shares of a large, established company. He will most likely make his purchase

A) in the primary market.

B) from an underwriter.

C) in the secondary market.

D) from the issuing corporation.

A

The correct answer is (C).
Newly issued shares are sold in the primary market, with the assistance of an underwriter. The purchase and sale of securities that have already passed through the primary market occurs in the secondary market. The shares of a large, established company have likely been through the primary market many years ago and are currently trading in the secondary market.

How well did you know this?
1
Not at all
2
3
4
5
Perfectly
4
Q

A stock market participant that buys shares of stock in a secondary market and holds shares as part of its inventory is most likely a(n)

A) stock exchange.

B) independent advisor.

C) broker-dealer.

D) self-regulatory organization.

A

The correct answer is (C).
Broker dealers serve two functions: to act as an intermediary in helping clients buy and sell securities and to generate a profit on the trading of undervalued securities. In their dealer function, broker-dealers can hold inventory when they believe the shares are undervalued with the intent of selling them at a later date when the price rises.

How well did you know this?
1
Not at all
2
3
4
5
Perfectly
5
Q

All of the following is (are) among the responsibilities of FINRA EXCEPT

A) educating investors.

B) fostering market transparency.

C) writing and enforcing rules governing the activities of all registered broker-dealer firms and registered brokers in the U.S.

D) writing and enforcing rules governing the activities of all registered investment advisors.

A

The correct answer is (D).
The SEC and state securities regulators are responsible for regulation of registered investment advisors.

How well did you know this?
1
Not at all
2
3
4
5
Perfectly
6
Q

Fred bought 100 shares of Apple at $115 per share. One year later, he sold the stock for $152 per share. During the year, Apple declared and paid dividends of $2 per share. What was Fred’s holding period return?

A) 32%

B) 34%

C) 73%

D) 206%

A

The correct answer is (B).
(Net proceeds + dividend – interest) ÷ equity invested = HPR
[($152 – $115) + $2] ÷ $115 = 33.91%

How well did you know this?
1
Not at all
2
3
4
5
Perfectly
7
Q

Which of the following can be greatly reduced by diversification?

A) Systematic risk

B) Market risk

C) Unsystematic risk

D) Systematic and unsystematic risk

A

The correct answer is (C).
Unsystematic risk is reduced by diversification.

How well did you know this?
1
Not at all
2
3
4
5
Perfectly
8
Q

Austin invested in the Very Value mutual fund 5 years ago. His returns were 25%, -5%, 10%, 0%, and 50%, respectively. What is the difference between the arithmetic average and the geometric average return over the 5 years?

A) 0%

B) 1.1%

C) 1.3%

D) 1.6%

A

The correct answer is (D).
Arithmetic average = (25%+-5%+10%+0%+50%)/5 = 16%
Geometric average =

  • [(1 + 25%) × (1 + -5%) × (1 + 10%) × (1 + 0%) × (1 + 50%)](1/ 5) – 1
  • [[1.9594] (ORANGE SHIFT YX ) 5 (ORANGE SHIFT 1/X)] - 1
  • 1.1440 - 1
  • 14.4%

Difference = 16% − 14.4% = 1.6%

How well did you know this?
1
Not at all
2
3
4
5
Perfectly
9
Q

Sam’s retirement fund is expected to earn a nominal rate of 7 percent, and the inflation rate is estimated at 3 percent. What is Sam’s real rate of return?

A) 1.43%

B) 2.33%

C) 3.88%

D) 4.00%

A

The correct answer is (C).
Real return = (1.07 ÷ 1.03) − 1 = 3.8835%

How well did you know this?
1
Not at all
2
3
4
5
Perfectly
10
Q

Security A has the following returns over 4 years: 4%, 7%, 0%, and -1%. What is the mean return and the standard deviation (sample) for Security A?

A) Mean of 2.5% and standard deviation of 3.2%

B) Mean of 2.5% and standard deviation of 3.7%

C) Mean of 4% and standard deviation of 3.2%

D) Mean of 4% and standard deviation of 3.7%

A

The correct answer is (B).
The mean is 2.5 percent and the standard deviation is close to 3.7 percent. You can calculate these using the ∑+ key on a financial calculator.

  1. 04 ∑+
  2. 07 ∑+
  3. 00 ∑+
    - 0.01 ∑+

{Orange] SxSy

How well did you know this?
1
Not at all
2
3
4
5
Perfectly
11
Q

If an investor owns a single share of a stock with a beta of 0.75, what can you conclude about his investment risk relative to the market?

A) The investor’s total risk is 3/4 of the risk of the market.

B) The investor is taking 75 percent more total risk than the risk of the market.

C) The investor’s systematic risk is ³⁄₄ of the risk of the market.

D) The investor’s diversifiable risk is 75% more than the total risk of the market.

A

The correct answer is (C).
Beta only measures systematic risk relative to 1, the market’s beta. Beta assumes a stock is added to an already diversified portfolio. Because a single stock has both diversifiable as well as systematic risk, an investor who purchases only a single stock may have a total risk greater than the market’s risk.

How well did you know this?
1
Not at all
2
3
4
5
Perfectly
12
Q

Ollie is considering two portfolios:

1) Portfolio A with a return of 10 percent and a standard deviation of 20 percent, and
2) Portfolio B with a return of 6 percent and a standard deviation of 8 percent.

Assuming the correlation between A and B is zero and Ollie invests 40 percent in A and 60 percent in B, what is the portfolio standard deviation?

A) 7.60%

B) 9.33%

C) 12.8%

D) 14.0%

A

The correct answer is (B).

  • Portfolio standard deviation = sqrt[(w1^2 x SD1^2) + (w2^2 x SD2^2)]
  • Portfolio standard deviation = sqrt[(0.4^2 x 20^2) + (0.6^2 x 8^2)]
  • Portfolio standard deviation = sqrt[(0.16 x 400) + (0.36 x 64)]
  • Portfolio standard deviation = sqrt(64 + 23.04) = sqrt(87.04) = 9.33
How well did you know this?
1
Not at all
2
3
4
5
Perfectly
13
Q

Which of the following is accurate regarding the capital market line (CML)?

A) The portfolios on the new efficient frontier are some combination of the risk-free asset and the market portfolio.

B) The CML is a line that begins at the risk-free rate of return and crosses the efficient frontier at the market portfolio.

C) The CML represents the most efficient portfolios of individual stocks.

D) The slope of the CML is an investor’s indifference curve.

A

The correct answer is (A).

The CML is a combination of the risk-free return and the market portfolio. Option (B) is incorrect, as the CML does not cross the efficient portfolio, rather it is tangent to it. Option (C) is incorrect as the CML consists of the market portfolio and the Rf. Option (D) is incorrect as the slope is the Sharpe Ratio.

How well did you know this?
1
Not at all
2
3
4
5
Perfectly
14
Q

Zoe has been saving diligently for many years and has accumulated an appropriate emergency fund along with substantial retirement savings that will allow her to enjoy all of her favorite activities during retirement and still make generous gifts to her nieces and nephews over the next several years. Zoe experiences extreme anxiety when her portfolio declines in value. Which of the following is true?

A) Zoe’s willingness (tolerance) to take on risk is greater than her ability (capacity) to take on risk, so her portfolio should have a moderately aggressive asset allocation.

B) Zoe’s ability (capacity) to take on risk is greater than her willingness (tolerance) to take on risk, so her portfolio should have a conservative asset allocation.

C) Zoe’s willingness (tolerance) to take on risk is greater than her ability (capacity) to take on risk, so her portfolio should have a conservative asset allocation.

D) Zoe’s ability (capacity) to take on risk is greater than her willingness (tolerance) to take on risk, so her portfolio should have a moderately aggressive asset allocation.

A

The correct answer is (B).
Risk capacity is a measurement of the amount of risk a client can afford to take on. Risk tolerance is the amount of risk a client is willing to take on. When risk capacity and risk tolerance are not in alignment, the more conservative of the two should determine the planning recommendations. In this case, Zoe has the ability to take on risk; however, since she lacks the tolerance (willingness) to take on risk, her portfolio should be invested with a more conservative asset allocation.

How well did you know this?
1
Not at all
2
3
4
5
Perfectly
15
Q

Which of the following techniques or strategies would take advantage of a perceived undervaluation in the energy sector of the economy?

A) Dollar-cost averaging into a portfolio

B) Strategic asset allocation rebalancing

C) Tactical asset allocation

D) Index funds

A

The correct answer is (C).
Tactical asset allocation is in response to short-term market conditions. Dollar-cost averaging is a method of investing over time. Strategic asset allocation is a long-term strategy, not one that takes advantage of sector imbalances.

How well did you know this?
1
Not at all
2
3
4
5
Perfectly
16
Q

The intersection of the security market line (SML) and the y axis occurs at the

A) market premium.

B) market portfolio.

C) real rate of return.

D) risk-free rate of return.

A

The correct answer is (D).
The SML is a line that connects the risk-free rate of return, which is on the y-axis, to the market portfolio.

How well did you know this?
1
Not at all
2
3
4
5
Perfectly
17
Q

The return from the CAPM is which of the following?

A) The risk-free return

B) The actual return

C) The risk-premium

D) The expected return

A

The correct answer is (D).
CAPM = Rf + β(Rm – Rf ) = the expected return based on beta, the market premium, and the risk-free rate of return

How well did you know this?
1
Not at all
2
3
4
5
Perfectly
18
Q

Consider this chart with the seven portfolios and the efficient frontier.

All of the following statements are correct EXCEPT:

A) Portfolio H does not exist in the real world.

B) Portfolio C dominates portfolios F and G.

C) Portfolios B, C, and D would be optimal portfolios for any investor.

D) Portfolio D is the riskiest option.

A

The correct answer is (C).
Option (C) is not correct; while it is true that these portfolios lie on the efficient frontier, the optimal portfolio for any given investor is one that both lies on the frontier and is appropriate for the investor’s risk capacity and risk tolerance. The other statements are correct. No portfolio may exist above the efficient frontier, so Portfolio H is unattainable. Portfolio C dominates F and G because it offers more return for the same level of risk. Portfolio D is the riskiest option because it lies furthest to the right.

How well did you know this?
1
Not at all
2
3
4
5
Perfectly
19
Q

All of the following statements regarding U.S. Treasury securities are correct EXCEPT:

A) Treasury bills (but not Treasury bonds or notes) are considered money market securities.

B) Interest income from Treasury securities is tax-free at the state level but not at the federal level.

C) All Treasury securities pay interest.

D) Treasuries with longer maturities tend to have higher yields.

A

The correct answer is (C).
Treasury bills do not pay interest – they are pure discount instruments.

How well did you know this?
1
Not at all
2
3
4
5
Perfectly
20
Q

All of the following regarding TIPS are true EXCEPT:

A) The interest paid on TIPS changes based on inflation and the changing principal amount of the TIPS.

B) TIPS are considered low-risk investments.

C) TIPS are most appropriate for young investors with many decades until retirement.

D) If deflation occurs over the life of the TIPS, the investor receives the greater of the principal amount or the par value.

A

The correct answer is (C).
TIPS are a low-risk, low-reward investment that only pays well in periods of high inflation. Young people are less exposed to inflation risk because their incomes and higher-risk investments will help keep pace with inflation. Older people into retirement, on the other hand, are very exposed to inflation risk, making TIPS a more appropriate investment for them.

How well did you know this?
1
Not at all
2
3
4
5
Perfectly
21
Q

Andy is considering purchasing a 12-year bond that is selling for $1,300. What is the yield to maturity (YTM) for this bond if it has an 8 percent coupon, paid semiannually?

A) 4.63%

B) 4.68%

C) 4.70%

D) 4.72%

A

The correct answer is (C).

P/YR 2

PV

($1,300)

N

24

Pmt

$40

FV

$1,000

i

4.70%

How well did you know this?
1
Not at all
2
3
4
5
Perfectly
22
Q

Assume that the 1-, 2-, 3-, 5-, 10-, 20-, and 30-year rates were 7 percent, 6.5 percent, 6 percent, 5.4 percent, 5.2 percent, 5.0 percent, and 4.8 percent, respectively. What type of yield curve is this?

A) Humped

B) Normal

C) Flat

D) Inverted

A

The correct answer is (D).
The yield curve is downward sloping, which is known as inverted.

How well did you know this?
1
Not at all
2
3
4
5
Perfectly
23
Q

Sam has a $3-million fixed-income portfolio that consists of Bond A, Bond B, Bond C, and Bond D. The bonds have durations of 2, 3, 8, and 10, respectively. If Sam has 20 percent invested in Bond A, 30 percent in Bond B, and 25 percent invested in each of the other two bonds, what is the duration for the portfolio? Assume that the correlation among the bonds is 0.5.

A) 5.50

B) 5.75

C) 5.80

D) 6.20

A

The correct answer is (C).
(0.2 × 2) + (0.3 × 3) + (0.25 × 8) + (0.25 × 10) = 0.4 + 0.9 + 2.0 + 2.5 = 5.8

How well did you know this?
1
Not at all
2
3
4
5
Perfectly
24
Q

A high-income investor owns a portfolio of bonds. Interest rates abruptly and sharply increased. Which of the following strategies would be most helpful in order to take advantage of the decrease in the value of this investor’s fixed-income securities?

A) The tax swap

B) The spread swap

C) The bullet strategy

D) The barbell strategy

A

The correct answer is (A).

As interest rates increase, the price of bonds decreases. This drop in value may create an opportunity to recognize the capital loss for tax purposes, which is the purpose of the tax swap.

How well did you know this?
1
Not at all
2
3
4
5
Perfectly
25
Q

The Anderson bond is a 5 percent coupon bond with semiannual coupon payments that matures in 10 years. If the YTM for this bond is 4 percent, what is the value of the bond?

A) $1,081.11

B) $1,081.76

C) $1,124.35

D) $1,125.03

A

The correct answer is (B).
P/YR 2

FV

$1,000.00

N

20

i

4.0%

Pmt

$25.00

PV

$1,081.76

26
Q

Which of the following statements is/are correct?

I. Longer maturity bonds are less volatile than shorter term bonds when interest rates fluctuate.
II. Higher coupon bonds are less volatile than lower coupon bonds when interest rates fluctuate.

A) I only

B) II only

C) Both I and II

D) Neither I nor II

A

The correct answer is (B).
When interest rates change, bonds with higher coupon payments and shorter maturities are less volatile.

27
Q

Byte, Inc., had net earnings of $2.5 billion last year. It has 500 million shares of common stock outstanding, which stayed the same since last year. It paid 50 cents per share per quarter this year as a dividend payment, which is the same as last year. This year’s earnings are 20% higher than last year. Which of the following is correct for this year?

A) The retention ratio equals 33.33%.

B) The retention ratio equals 50.00%.

C) The payout ratio equals 33.33%.

D) The payout ratio equals 40.00%.

A

The correct answer is (C).

  • The dividend per share equals $2.00.
  • EPS equals $5.00 last year, which is found by dividing net earnings by outstanding shares.
    • 2,500,000,000 / 500,000,000
  • This year’s EPS is 20% higher or $6.00.
    • 3,000,000,000 / 500,000,000
  • The payout ratio = dividend / EPS
    • $2.00 / $6.00 = 33.33%
28
Q

$100 of stock is purchased with a 50% initial margin and a 30% maintenance margin. At what price would a margin call occur?

A) $28.58

B) $70.00

C) $71.42

D) $85.00

A

The formula used to determine the price below when a margin call will occur is: debt / (1 − MMR).

  • A margin call if the price falls below $50 / (1 − 0.30) = $71.42.
  • The price drop is $28.58, or 1 − debt / (1 − MMR).= 1 − (0.50 / 0.70) = 28.58%.
29
Q

Juan bought XYZ Company stock at $40 per share. Today, the stock sells for $52 per share. Juan likes the long-term prospects for XYZ stock but wants some protection against price decreases. Which of the following orders is the best way for Juan to both participate in future price increases and ensure a minimum profit of $6 per share?

A) A limit order to sell at $46

B) A stop-limit order, stop price = $47, limit price = $46

C) A limit order to buy at $40

D) A stop-sell order, stop price = $46

A

The correct answer is (B).
A limit order will not work as it is a sell order at or above $46 that would be executed today. It does not provide the downside protection Juan is looking for. The stop-limit order will place a limit order with a price of $46 once the stock reaches $47. The stop-sell order results in a market order once the stock price reaches $46, which likely will result in the order being filled below $46.

30
Q

Preferred stock is a hybrid security. In which ways is preferred stock like common stock?

A) Representation of ownership interest

B) Guaranteed dividend payments

C) Premiums attached to its par value

D) Payment of interest

A

The correct answer is (A).
Preferred stock represents an equity ownership position. Preferred shareholders typically receive a fixed dividend (not interest) payment, although no dividend payments are ever guaranteed. Par value is independent of the notion of a hybrid security.

31
Q

Examples of income stocks are least likely to include stock categories:

A) Tech

B) Blue chip

C) Utility

D) Financial institutions

A

The correct answer is (A).
Tech stocks typically are high growth firms that require any cash flows to be reinvested back into the business, leaving little for the payment of dividends. Blue chips, utilities, and financial institutions generally pay dividends, often even high dividends.

32
Q

Which of the following statements regarding the primary market is (are) correct?

I. Investment bankers most commonly use best efforts to facilitate primary offerings.
II. Initial public offerings are a significant component of the primary market.

A) I only

B) II only

C) Both I and II

D) Neither I nor II

A

The correct answer is (B).
Statement I is incorrect because investment bankers most commonly use a firm commitment to facilitate a primary offering.

33
Q

Onion, a large technology company, has stock that pays a $1 dividend semiannually. Its current earnings per share is $10. If the stock is currently trading at $120, what is the dividend yield percentage?

A) 0.83%

B) 1.67%

C) 10.00%

D) 20.00%

A

The correct answer is (B).
The dividend yield percentage is equal to the total annual dividends per share divided by the stock price. ($1 × 2) / $120 = 1.67%

34
Q

National Fund is a closed-end fund with a cash balance of $80 million. American Fund is an open-end fund with a cash balance of $300 million. Both National and American have $2 billion in market value of assets. National has less cash most likely because it

A) invests in less liquid assets than American.

B) does not permit investors to redeem shares at any time.

C) has greater management fees than American.

D) has less access to capital than American.

A

The correct answer is (B).
Open-end funds must have much more cash on hand than closed-end funds because their shareholders can redeem shares daily, and those transactions occur at the closing net asset value of the fund. Closed-end fund shareholders must sell their shares on an exchange to redeem them, thus requiring closed-end funds to have minimal cash on hand.

35
Q

Kay Fields compares the fees and expenses of several equity funds. Fields will likely have the most difficulty in evaluating

A) trading costs.

B) the 12b-1 fees.

C) the back-end loads.

D) the front-end loads.

A

The correct answer is (A).
Trading costs are the most difficult for an investor to assess, partly because the other fees and expenses are disclosed to investors in the prospectus, but also because some trading costs are implicit. Examples of implicit trading costs are bid-ask spreads, market-impact costs, and opportunity costs.

36
Q

Which of the following statements about mutual fund loads is correct?

A) A back end load may be charged when an investor redeems shares of a mutual fund shortly after buying them.

B) A no-load mutual fund will have either no front-end load or no back-end load.

C) Loads are considered transaction costs and do not appear in the fund’s prospectus.

D) Back-end loads are included in a fund’s annual expense ratio until the load is paid off.

A

The correct answer is (A).
Back-end loads are typically charged when an investor redeems shares of a mutual fund shortly after buying those shares. Statement (B) is incorrect because a no-load fund will have neither a front-end nor a back-end load. Statement (C) is incorrect because loads are not considered transaction costs and will appear in a fund’s prospectus. Statement (D) is incorrect because back-end loads are not included in the expense ratio and will never be paid unless the investor redeems shares.

37
Q

Which type of mutual fund share generally converts to an “A share” after a period of time?

A) Class B shares

B) Class C shares

C) Both B and C shares

D) Neither B nor C shares

A

The correct answer is (A).
Most Class B shares have a conversion feature that allows the shares to convert to Class A after a specified time period. Only in certain cases may Class C shares convert to A shares.

38
Q

Which of the following statements concerning a unit investment trust is correct?

A) It has an unmanaged portfolio.

B) It usually consists of common stocks.

C) It is most appropriate for young people.

D) It requires constant management attention.

A

The correct answer is (A).
Option (B) is incorrect as UITs generally consist of bonds. Option (C) is incorrect as UITs are better for generating income, not the typical objective of young people. Option (D) is incorrect as the fund is generally passive.

39
Q

Which of the following is an advantage of mutual funds over ETFs?

A) Lower stock turnover

B) Lower average expenses

C) Ability to redeem at net-asset value

D) Greater tax efficiency

A

The correct answer is (C).
Mutual fund shares can be redeemed for net-asset value while ETFs can be sold at market value, which will be very close to net-asset value if the shares are highly liquid. ETFs generally are more tax efficient because they tend to have lower stock turnover, and on average, have lower expenses.

40
Q

Index funds are most notable for which one of the following features?

A) Low management fees

B) Outperformance of the market

C) Liquidation during open-market hours

D) Trading on exchanges

A

The correct answer is (A).
Index fund managers do not perform significant levels of research on security selection and allocation, which tends to reduce management fees. Index funds are among the funds with the lowest expenses and fees. Option (B) is incorrect because index funds do not tend to outperform the market; rather, they tend to mimic the market. Option (C) is incorrect because ETFs, not index funds, may be liquidated during open-market hours. Option (D) is incorrect because closed-ended funds and ETFs are traded on exchanges; index funds are bought and sold directly from the fund manager.

41
Q

As of 2021, what is the maximum possible federal tax rate applied to capital gains from the sale of common stock?

A) 18.8%

B) 20.0%

C) 23.8%

D) 28.0%

A

The correct answer is (C).

The maximum possible capital-gains rate is the base 20% maximum plus the 3.8% net investment income tax for a total of 23.8%.

42
Q

Which of the following regarding futures contracts is least accurate?

A) Futures contracts are less liquid than forward contracts.

B) Futures contracts are marked to market.

C) Futures contracts are traded on a regulated exchange.

D) Futures contracts allow more delivery options than forward contracts.

A

The correct answer is (A).
Futures contracts are more liquid than forward contracts, because they are traded on organized exchanges and are standardized. Forward contracts are customized to the parties involved in the transaction.

43
Q

A contract that is an agreement for immediate exchange of funds for assets is called a

A) futures contract.

B) forward contract.

C) call option contract.

D) spot contract.

A

The correct answer is (D).
A spot contract is a contract that calls for immediate delivery of goods. The spot price or rate is the current price or rate.

44
Q

A corn farmer who wants to hedge the price of corn should enter into what type of contract?

A) Buy a corn futures contract.

B) Sell a corn futures contract.

C) Long position in a corn call-option contract.

D) Long position in a commodity put-option index.

A

The correct answer is (B).
A corn farmer would enter into a short (sell) futures contract, because he wants to sell his corn.

45
Q

All of the following are correct regarding derivative securities EXCEPT:

A) Derivatives can be used for speculation and hedging.

B) The use of leverage is both an advantage and a disadvantage of derivatives.

C) The value of a derivative is tied to the value of an underlying security or asset.

D) Most types of derivatives are suitable investments for unsophisticated investors.

A

The correct answer is (D).
The use of leverage exaggerates both gains and losses, which can be an advantage or a disadvantage. This exaggeration of gains and losses, along with a derivative’s relative complexity, makes most types of derivatives unsuitable for small, unsophisticated investors.

46
Q

An “in the money put”

A) has an intrinsic value < 0.

B) is where the stock price > strike price.

C) is where the stock price < strike price.

D) has a time value > 0.

A

The correct answer is (C).
The stock price is less than the strike price.

47
Q

Which of the following statements concerning convertible bonds is (are) correct?

I. Convertible bonds provide an inexpensive way to buy stock.
II. Convertible bonds provide a lower yield than do straight bonds of comparable credit risk and maturity because of the attractiveness of the conversion feature.

A) I only

B) II only

C) Both I and II

D) Neither I nor II

A

The correct answer is (B).
Statement I is incorrect because convertible bonds normally trade at a premium-to-conversion value. Thus, buying a convertible bond and then converting the bond provides an investor with fewer shares of stock than if the investor had purchased the stock outright directly at the start.

48
Q

Colin purchased Apple stock earlier this year. The stock has increased significantly. While Colin likes the company’s long-term prospects, he is concerned about a rapid price drop. Which of the following strategies would protect him from such a price drop?

A) Buy Apple puts.

B) Sell Apple puts.

C) Buy Apple calls.

D) Sell Apple calls.

A

The correct answer is (A).
A put option protects against drops in Apple stock price.

49
Q

An investor buys a call option on ABC stock with an exercise price of $45 for $5 per share. When ABC increases to $50, the call is exercised, and the stock is immediately sold. What is the result?

A) A loss of $500

B) No gain or loss

C) A profit of $500

D) A profit of $1,000

A

The correct answer is (B). 
($5 gain from stock price × 100) − ($5 premium × 100) = $0

50
Q

Which of the following investing risks is most significant for real estate investors as compared to investors in publicly traded stocks?

A) Market risk

B) Default risk

C) Liquidity risk

D) Idiosyncratic risk

A

The correct answer is (C).
Liquidity risk is far higher in real estate investing than in trading in stock that can be sold on a secondary market exchange. Stock investors are subject to market risk, idiosyncratic risk, and the risk that the firm will declare bankruptcy.

51
Q

Philip buys undeveloped beachfront property. He is subject to all of the following risks EXCEPT

A) Market risk

B) Default risk

C) Liquidity risk

D) Political risk

A

The correct answer is (B).
Philip, as an investor in real estate, is subject to market risk, liquidity risk and political risk. He is not subject to default risk. There is no indication that there is debt involved in this question. In addition, he would not be the one subject to default risk even if debt was involved in the purchase of the real estate.

52
Q

All of the following positions are examples of the types of holdings found in a hedge fund manager’s portfolio EXCEPT

A) the long-only position in a gasoline futures contract.

B) a put option written on a barrel of oil.

C) long and short positions in the equity of two industry leaders.

D) the long position only in a highly rated municipal bond.

A

The correct answer is (D).
Hedge funds rarely invest in investment grade municipal bonds, because the risk is generally very low and the return is not sufficient to warrant interest by the hedge fund manager. Hedge funds often invest in options and futures contracts and also try to benefit when one industry leader outperforms its main rival.

53
Q

Two years ago, Isabella purchased 100 shares of CVC, Inc., for $15,000. Unfortunately, the value of the shares has dropped to $10,000. Isabella’s daughter, Shea, was heading off to college, and Isabella was tired of waiting for a return on the stock. Isabella gave the stock to Shea when it was worth $10,000 to help fund Shea’s education. If Shea sells the shares for $12,000 three months after the transfer, what is the amount and character of her gain or loss?

A) $0

B) $2,000 short-term capital gain

C) $3,000 long-term capital loss

D) $3,000 short-term capital loss

A

The correct answer is (A).
Isabella gave loss property to her daughter. When a taxpayer transfers property with a loss to someone else, and the recipient sells the property for an amount between the donor’s adjusted basis and the fair market value of the stock on the date of the gift, no gain or loss is recognized. In this example, the no-gain, no-loss corridor is from $10,000 (the fair market value of the stock on the date of the gift) to $15,000 (the adjusted basis in the hands of the donor). If Shea sold the stock for any amount between $10 and $15 thousand dollars, she is not required to recognize any gain, and she is prohibited from recognizing any loss on the transaction. Since there is no gain or loss, there is no need to categorize the tax result as either long or short term.

54
Q

Which of the following is a violation of the wash-sale rule?

A) You sell 27 shares of an S&P 500 index fund at a loss. Then 15 days later, you buy 42 shares of a Russell 3000 index fund.

B) You have brokerage accounts at two firms. At the first firm, you buy 100 shares of PQR stock. Then 22 days later, you sell 100 shares of PQR stock, bought the prior year, for a loss in your account at the other firm.

C) Your spouse buys 200 shares of UK stock. Then 31 days later, you sell 100 shares of the same stock, bought 43 days earlier, at a slight loss.

D) You sell 100 shares of UK stock at a slight gain. Then 4 days later, your spouse buys 200 shares of the same stock.

A

The correct answer is (B).
Option (A) is incorrect because the two indices are sufficiently different. Option (C) is incorrect because the purchase is more than 30 days prior to the sale. Option (D) is incorrect because the stock is sold at a gain.

55
Q

Higgins purchased 2,000 shares of Dunlap, Inc., stock for $28 per share three years ago and paid a $50 brokerage commission on the transaction. This year he sold the shares for $37 per share and paid a $50 brokerage commission on the transaction. What is the amount of his taxable gain or loss?

A) $0

B) $17,900

C) $18,000

D) $74,000

A

The correct answer is (B).
The amount realized from the sale is $74,000 − $50 commission = $73,950. His cost basis is $56,000 + $50 commission = $56,050. The taxable gain is $73,950 − $56,050 = $17,900.

56
Q

Cody purchased 400 shares of NAY stock six years ago when it was trading at $65 per share. Unfortunately, NAY has been steadily declining. Cody sold his shares this year for $18 per share. This year Cody also sold 800 shares of a mutual fund that he purchased 6 months ago. His average cost per share was $15 and he sold the shares for $32. Assuming Cody had no other capital transactions this year, what is Cody’s tax consequence from these transactions?

A) $5,200 long-term loss this year

B) $18,800 long-term loss, $13,600 short-term gain taxed at ordinary income rates this year

C) $3,000 ordinary loss this year with a $15,800 ordinary loss carried forward up to 5 years

D) $3,000 ordinary loss this year with a $2,200 loss carried forward indefinitely

A

The correct answer is (D).
The NAY stock sale resulted in a long-term loss of $18,800 ($47 loss per share × 400 shares). The mutual fund sale resulted in a short-term gain of $13,600 ($17 gain × 800 shares). The entire $13,600 gain is offset by the loss, leaving a net long-term loss of $5,200. Cody can take $3,000 of the loss against ordinary income this year, and the remaining loss is carried forward indefinitely (until death).

57
Q

All the following statements regarding investment strategies for tax-advantaged accounts are correct EXCEPT:

A) The general rule is that one should place securities with interest income in qualified accounts and securities with good prospects for capital gains in nonqualified accounts.

B) The major tax-advantaged accounts include IRAs and qualified plans through work such as the 401(k).

C) Investors should always maximize their tax-advantaged accounts before contributing to taxable brokerage accounts.

D) All tax-advantaged accounts allow investors to defer the recognition of taxable income and some even allow for tax-free growth.

A

The correct answer is (C).

There are cases in which investors should prioritize contributing to taxable brokerage accounts (e.g., when creating an emergency fund or a down payment fund). Tax-advantaged accounts generally limit withdrawals, which may subject the investor to additional taxes or penalties if the funds are used for certain nonqualified purposes.

58
Q

An asset-allocation strategy that periodically purchases assets that have fallen in value and sells assets that have risen in value is

A) a buy-and-hold strategy.

B) a tactical allocation strategy.

C) a dynamic allocation strategy.

D) a constant-weighting strategy.

A

The correct answer is (D).
A constant-weighting strategy returns the weights to their original strategic values. Consequently, when the price of a security falls, its weights also fall. The investor is forced to purchase additional shares to increase the allocation.

59
Q

A real estate investor bought a property for $150,000 and rented it out over 3 years before selling it for $160,000. His income and expenses over this period are shown in the table below. Based on this information, what was his dollar-weighted return?

YearIncomeExpenses

1$7,000-$10,000

2$9,000-$10,000

3$10,000-$1,000

A) 1.39%

B) 2.40%

C) 3.18%

D) 3.33%

A

First, calculate each time period’s total cash inflows and outflows:

Period Cash Flow

  • -150,000
  • -3,000
  • -1,000
  • 169,000

Then, to solve this problem using an HP10bII+, use the following keystrokes:

150,000

[+/-]

[CFj]

3,000

[+/-]

[CFj]

1,000

[+/-]

[CFj]

169,000

[CFj]

[SHIFT]

[IRR/YR]

The correct answer is 3.1809, or about 3.18%.

60
Q

Which of the following is (are) correct with respect to bond portfolio strategies?

I. A portfolio that has 40% invested in bonds with a 2-year maturity and the other 60% invested in bonds with a 10-year maturity is known as a bullet portfolio.

II. If the portfolio’s duration statistic matches the intended holding period, then the portfolio is always referred to as a laddered portfolio.

A) I only

B) II only

C) Both I and II

D) Neither I nor II

A

The correct answer is (D).
The portfolio in Statement I is an example of a barbell portfolio. The portfolio in Statement II is an example of an immunized portfolio.

61
Q

A writer of a put or call may have tax consequences in all the following situations EXCEPT

A) when the option premium is received.

B) when the option is exercised.

C) when the option is expires.

D) when the option position is closed.

A

The correct answer is (A).
There are no tax consequences when an option is initially written. Tax consequences only arise when the option is exercised, when the option expires, or when the option position is closed with an offsetting trade.

62
Q
A